Answer (B) is correct . EOQ= Sq.root [(2* variable order cost * periodic demand) / unit carrying cost] = Sq.root [(2 * $10 * 1,000 units) / ($10* 20%)] = Sq.root ($20,000/ $2) = Sq.root (10,000) = 100 u Answer (A) is incorrect because The amount of 45 units results in a higher total cost (carrying costs plus ordering costs) than quantities of 100. Answer (C) is incorrect because The amount of 141 units results in a higher total cost (carrying costs plus ordering costs) than quantities of 100. Answer (D) is incorrect because The amount of 1,000 units results in a higher total cost (carrying costs plus ordering costs) than quantities of 100.
|